How to Optimize Cone Volume with Given Slant Height?

  • Thread starter Thread starter girlygirl93
  • Start date Start date
  • Tags Tags
    Calculus Cone
Click For Summary
To optimize the volume of a cone with a fixed slant height, the relationship between the radius (r), height (h), and slant height (l) must be established using the equation l^2 = r^2 + h^2. The volume of the cone is given by V = (1/3)πr^2h. By substituting r^2 from the slant height equation into the volume equation, the volume can be expressed solely in terms of height, V(h). Differentiating this new volume function with respect to h allows for finding the maximum volume while keeping the slant height constant. This approach effectively combines geometry and calculus to achieve the desired optimization.
girlygirl93
Messages
5
Reaction score
0
Hello there :) I'm having tons of trouble figuring out how to finish this problem.
A cone is to be constructed having a given slant height of l>0 . Find the radius and height which give maximal volume.



I am unsure of which variables to keep in order for it to be maximized, and how to go about optimizing it.



This is how I was going about it: I think that the cross-section of the cone makes a right angled triangle, for which the equation would be l^2= b^2 + h^2, and in order to maximize the volume you must relate it to the volume equation V = 1/3(pi)r^2h, but I am having trouble putting it together, to be able to differentiate and then maximize.
 
Physics news on Phys.org
ok so colume as a function of r & h is
V(r,h) = 1/3(pi)r^2h

but you also know (assuming b=r)
l^2=h^2+r^2

rearranging the contsrtaint gives
r^2 = h^2-l^2

and you can subsitute into you volume equation, to get V(h) only. Then you can differentiate w.r.t. h and maximise remembering that l is constant
 
Question: A clock's minute hand has length 4 and its hour hand has length 3. What is the distance between the tips at the moment when it is increasing most rapidly?(Putnam Exam Question) Answer: Making assumption that both the hands moves at constant angular velocities, the answer is ## \sqrt{7} .## But don't you think this assumption is somewhat doubtful and wrong?

Similar threads

Replies
9
Views
4K
  • · Replies 9 ·
Replies
9
Views
2K
Replies
7
Views
4K
  • · Replies 17 ·
Replies
17
Views
5K
Replies
5
Views
2K
  • · Replies 4 ·
Replies
4
Views
5K
  • · Replies 2 ·
Replies
2
Views
2K
  • · Replies 1 ·
Replies
1
Views
2K
Replies
7
Views
7K
  • · Replies 5 ·
Replies
5
Views
5K